Question

In: Advanced Math

If V = U ⊕ U⟂ and V = W ⊕ W⟂, and if S1: U...

If V = U ⊕ U and V = W ⊕ W, and if S1: U → W and S2: U → W are isometries, then the linear operator defined for u1 ∈ U and u2 ∈ U by the formula S(u1 + u2) = S1u1 + S2u2 is a well-defined linear isometry. Prove this.

Solutions

Expert Solution


Related Solutions

Prove that if U, V and W are vector spaces such that U and V are...
Prove that if U, V and W are vector spaces such that U and V are isomorphic and V and W are isomorphic, then U and W are isomorphic.
Let W denote the set of English words. For u, v ∈ W, declare u ∼...
Let W denote the set of English words. For u, v ∈ W, declare u ∼ v provided that u, v have the same length and u, v have the same first letter and u, v have the same last letter. a) Prove that ∼ is an equivalence relation. b) List all elements of the equivalence class [a] c) List all elements of [ox] d) List all elements of [are] e) List all elements of [five]. Can you find more...
Let V be a vector space and let U and W be subspaces of V ....
Let V be a vector space and let U and W be subspaces of V . Show that the sum U + W = {u + w : u ∈ U and w ∈ W} is a subspace of V .
True/False Question: If sppan{u,v}=W where u not equal to v. Then dim(W)=2. Answer: False Reasoning let...
True/False Question: If sppan{u,v}=W where u not equal to v. Then dim(W)=2. Answer: False Reasoning let u=1, v=2 then span(1,2}=R but dim(R)=1 not 2. I know the answer is false. please tell me whether my reasoning is correct.
Let A ∈ L(U, V ) and B ∈ L(V, W). Assume that V is finite-dimensional....
Let A ∈ L(U, V ) and B ∈ L(V, W). Assume that V is finite-dimensional. Let X be a 3 × 5 matrix and Y be a 5 × 2 matrix. What are the largest and smallest possible ranks of X, Y, and XY? Give examples of the matrix to support your answers
3.5.4 ([Ber14, Ex. 3.6.14]). Let T : V → W and S : W → U...
3.5.4 ([Ber14, Ex. 3.6.14]). Let T : V → W and S : W → U be linear maps, with V finite dimensional. (a) If S is injective, then Ker ST = Ker T and rank(ST) = rank(T). (b) If T is surjective, then Im ST = Im S and null(ST) − null(S) = dim V − dim W
Let X ∈ L(U, V ) and Y ∈ L(V, W). You may assume that V...
Let X ∈ L(U, V ) and Y ∈ L(V, W). You may assume that V is finite-dimensional. 1)Prove that dim(range Y) ≤ min(dim V, dim W). Explain the corresponding result for matrices in terms of rank 2) If dim(range Y) = dim V, what can you conclude of Y? Give some explanation 3) If dim(range Y) = dim W, what can you conclude of Y? Give some explanation
Let V be a vector space, and suppose that U and W are both subspaces of...
Let V be a vector space, and suppose that U and W are both subspaces of V. Show that U ∩W := {v | v ∈ U and v ∈ W} is a subspace of V.
Let u and v be orthogonal vectors in R3 and let w = 3u + 6v....
Let u and v be orthogonal vectors in R3 and let w = 3u + 6v. Suppose that ||u|| = 5 and ||v|| = 4. Find the cosine of the angle between w and v.
1. Let U = {r, s, t, u, v, w, x, y, z}, D = {s,...
1. Let U = {r, s, t, u, v, w, x, y, z}, D = {s, t, u, v, w}, E = {v, w, x}, and F = {t, u}. Use roster notation to list the elements of D ∩ E. a. {v, w} b. {r, s, t, u, v, w, x, y, z} c. {s, t, u} d. {s, t, u, v, w, x, y, z} 2. Let U = {r, s, t, u, v, w, x, y, z},...
ADVERTISEMENT
ADVERTISEMENT
ADVERTISEMENT